Show that transformation is Canonical

In summary, in this conversation, the speakers discuss a problem involving a given coordinate transformation and the Hamiltonian. They mention the condition for a transformation to be canonical and discuss how to check if the given transformation is canonical using Poisson's brackets. They also discuss the relationship between a generating function and a canonical transformation. The conversation ends with a question about partial derivatives and a reference to a book that may be helpful.
  • #1
Gunthi
65
1

Homework Statement


Given the transformation [tex]Q=qe^{\gamma t}[/tex] and [tex]P=pe^{-\gamma t}[/tex] with the Hamiltonean [tex]H=\frac{p^2e^{-2\gamma t}}{2m}+\frac{m\omega^2q^2e^{2\gamma t}}{2}[/tex] show that the transformation is Canonical


Homework Equations



I know that the condition for a transformation to be canonical is [tex]\sum_i p_i\dot{q_i}-P_i\dot{Q_i}=H-K+\frac{dF_2}{dt}[/tex] but don't know what to do with it...

I've managed to prove from [tex]\frac{dF_2}{dq_i}=p_i[/tex] and [tex]\frac{dF_2}{dP_i}=Q_i[/tex] that [tex]F_2=qPe^{\gamma t}[/tex].
 
Last edited:
Physics news on Phys.org
  • #2
I have another question related to this problem. Does having the generating function guarantee that the coordinate transformation associated with it is canonical? Or is it a necessary but not sufficient condition?
 
  • #3
I know a way to check out whether the transformation is canonical but it doesn't involve the Hamiltonian.
If you know Poisson's brackets, if you can show that [itex][Q,P]_{q,p}=1[/itex] then you'd be done.
To answer your question "Does having the generating function guarantee that the coordinate transformation associated with it is canonical? Or is it a necessary but not sufficient condition?", I'm not 100% sure but I'd say yes (99% sure).
 
  • #4
I've been messing around with the coordinates and got this:

If we have a general invertible transformation of the type [tex]Q=Q(q,p,t)\Leftrightarrow q=q(Q,P,t)[/tex] and [tex]P=P(q,p,t)\Leftrightarrow p=p(P,Q,t)[/tex] then the following is true:

\begin{matrix}
\dot{q}=\frac{\partial q}{\partial Q}\dot{Q}+\frac{\partial q}{\partial P}\dot{P}\\
\dot{p}=\frac{\partial p}{\partial Q}\dot{Q}+\frac{\partial p}{\partial P}\dot{P}
\end{matrix}

From Hamilton's eqs we have (inserting the given coordinate transformations):
\begin{matrix}
\dot{q}=\frac{\partial H}{\partial p}=\frac{\partial H}{\partial P}e^{-\gamma t}=\frac{\partial q}{\partial Q}\dot{Q}\Rightarrow \frac{\partial H}{\partial P}=\dot{Q}\\
\dot{p}=-\frac{\partial H}{\partial q}= -\frac{\partial H}{\partial Q}e^{\gamma t}=\frac{\partial p}{\partial P}\dot{P}\Rightarrow -\frac{\partial H}{\partial Q}=\dot{P}
\end{matrix}

The form of Hamilton's equations is the same but from what I've seen in Goldstein, when one makes a canonical transformation the form is preserved not for the initial Hamiltonian but for the "Kamiltonian" [tex]K=H+\frac{dF_2}{dt}[/tex] i.e.

\begin{matrix}
\frac{\partial K}{\partial P}=\dot{Q}\\
-\frac{\partial K}{\partial Q}=\dot{P}
\end{matrix}

and because I've already proved that there exists a function [itex]F_2[/itex] I get the feeling that the proof is not correct. What am I doing wrong here?

fluidistic said:
I know a way to check out whether the transformation is canonical but it doesn't involve the Hamiltonian.
If you know Poisson's brackets, if you can show that [itex][Q,P]_{q,p}=1[/itex] then you'd be done.
To answer your question "Does having the generating function guarantee that the coordinate transformation associated with it is canonical? Or is it a necessary but not sufficient condition?", I'm not 100% sure but I'd say yes (99% sure).

Thanks for the advice! Actually, calculating with the Poisson brackets is another part of this problem, but instead it is asked to evaluate [itex][Q,Q],[P,P],[Q,P],[q,H],[p,H][/itex] without explicit calculation.

For the first two the answer is obviously zero because of the crossed derivatives, the third one, being this a suposedly canonical transformation is 1 and for the last two I'm having a little bit of a problem because if I use [itex]\frac{du}{dt}=[u,H]+\frac{\partial u}{\partial t}[/itex] on q and p I am not sure if it is [itex]\frac{dq}{dt}[/itex] or [itex]\frac{\partial q}{\partial t}[/itex] (for p too) that is zero. I know, this is a dumb question, but when a function depends only on t does the partial derivative looses its meaning?
 
  • #5
Ok, since you found out [itex]F_2[/itex] then I think you are done. Let me give you a useful link: http://books.google.com.ar/books?id...a transformation is canonical poisson&f=false.
About the Poisson's bracket question, you are right for the 3 first. As for the last 2 I must say I'm not sure; I'd consult Landau's book on classical mechanics first and then other books.
About your question about partial derivatives; when you have a function of only 1 variable a partial derivative is the same as a total derivative.
But I doubt q only depends on t.
 
  • Like
Likes chikou24i
  • #6
fluidistic said:
Ok, since you found out [itex]F_2[/itex] then I think you are done. Let me give you a useful link: http://books.google.com.ar/books?id...a transformation is canonical poisson&f=false.
About the Poisson's bracket question, you are right for the 3 first. As for the last 2 I must say I'm not sure; I'd consult Landau's book on classical mechanics first and then other books.
About your question about partial derivatives; when you have a function of only 1 variable a partial derivative is the same as a total derivative.
But I doubt q only depends on t.

Thanks for the book! I tried to see if there was anything in Landau that could help but didn't find it yet. Anyhow, I found this http://solar.physics.montana.edu/dana/ph411/p_brack.pdf and I can't figure out why, in page 2 after equation (3) [itex]\frac{\partial q_1}{\partial t}=0[/itex]. Isn't [itex]q_1[/itex] also a function of t? Why isn't [itex]\frac{dq_1}{dt}[/itex] also zero?
 
  • #7
Gunthi said:
Thanks for the book! I tried to see if there was anything in Landau that could help but didn't find it yet. Anyhow, I found this http://solar.physics.montana.edu/dana/ph411/p_brack.pdf and I can't figure out why, in page 2 after equation (3) [itex]\frac{\partial q_1}{\partial t}=0[/itex]. Isn't [itex]q_1[/itex] also a function of t? Why isn't [itex]\frac{dq_1}{dt}[/itex] also zero?
I'm in the same boat as you. I do not understand why it is so.
For sure, [itex]q_1[/itex] isn't a constant of motion, otherwise we'd have [itex]\{ q_1 , H \}=0[/itex]. So there's indeed a dependency of [itex]q_1[/itex] with respect to time. Thus, I do not understand how it's possible that [itex]\frac{\partial q _1 }{\partial t } =0[/itex].
I wish someone could enlighten us!
 
  • #8
I'm just posting this so that the thread doesn't seem closed.
 

What is transformation in a scientific context?

In a scientific context, transformation refers to a process of changing or converting something into a different form or state. This can occur on a molecular, cellular, or even societal level.

What does it mean for a transformation to be canonical?

A canonical transformation is a type of mathematical transformation that preserves the canonical structure of a physical system. This means that the equations of motion and the fundamental physical laws remain unchanged under the transformation.

How is the canonical structure of a physical system determined?

The canonical structure of a physical system is determined by the symplectic structure, which is a mathematical structure that describes the phase space of a system. The symplectic structure is defined by the Poisson bracket, which is a mathematical operation that determines the evolution of physical quantities.

Why is it important to show that a transformation is canonical?

Showing that a transformation is canonical is important because it provides a mathematical proof that the fundamental laws of physics are preserved. This allows for a better understanding and analysis of physical systems, and can also help to identify new symmetries and conserved quantities.

What are some examples of canonical transformations?

Some examples of canonical transformations include rotations and translations in classical mechanics, conformal transformations in electromagnetism, and Lorentz transformations in special relativity. These transformations preserve the symplectic structure and therefore maintain the fundamental laws of physics.

Similar threads

  • Advanced Physics Homework Help
Replies
9
Views
1K
  • Advanced Physics Homework Help
Replies
1
Views
2K
  • Advanced Physics Homework Help
Replies
1
Views
681
  • Advanced Physics Homework Help
Replies
2
Views
225
Replies
19
Views
1K
  • Advanced Physics Homework Help
Replies
3
Views
791
Replies
7
Views
5K
Replies
7
Views
1K
  • Advanced Physics Homework Help
Replies
4
Views
1K
  • Advanced Physics Homework Help
Replies
3
Views
1K
Back
Top